Which statement is true of the function flx) = -3/x? Select three options. The function is always increasing. The function has a domain of all real numbers. The function has a range of tyl-

Which Statement Is True Of The Function Flx) = -3/x? Select Three Options. The Function Is Always Increasing.

Answers

Answer 1

Notice that the given function:

1.- has a domain of all real numbers because you can always compute the cube root for all real numbers,

2.- the function has a range of all real numbers, and

3.- the function is a reflection of

[tex]y=\sqrt[3]{x}[/tex]

over the x-axis.

Answer: Options 2, 3, and 4.


Related Questions

Convert the following mixed number to improper fraction10 /2/57 3/20

Answers

The question asks us to convert mixed fractions to improper fraction.

The first Mixed fraction is:

[tex]\begin{gathered} 10\frac{2}{5} \\ \text{whole number = 10} \\ \text{ numerator = 2} \\ \text{ denominator = 5} \end{gathered}[/tex]

In order to convert this into an improper fraction, we need to follow some steps:

1. Multiply the denominator by the whole number.

2. Add the result of the multiplication in step 1 to the numerator.

3. The result from step 2 is the new numerator and use the current denominator as the new denominator.

Let us now apply these steps to answer the question

1. Multiply the denominator by the whole number.

[tex]5\times10=50[/tex]

2. Add the result of the multiplication in step 1 to the numerator.

[tex]50+2=52[/tex]

3. The result from step 2 is the new numerator and use the current denominator as the new denominator.

[tex]\begin{gathered} \text{new numerator = 52} \\ \text{new denominator = 5} \\ \therefore\frac{52}{5} \end{gathered}[/tex]

Therefore, the answer is:

[tex]10\frac{2}{5}=\frac{52}{5}[/tex]

Now, let us use the same rules for the next question.

[tex]7\frac{3}{20}[/tex]

1. Multiply the denominator by the whole number.

[tex]7\times20=140[/tex]

2. Add the result of the multiplication in step 1 to the numerator.

[tex]140+3=143[/tex]

3. The result from step 2 is the new numerator and use the current denominator as the new denominator.

[tex]\begin{gathered} \text{new numerator= 143} \\ \text{new denominator = 20} \\ \therefore\frac{143}{20} \end{gathered}[/tex]

Therefore, the final answer is:

[tex]7\frac{3}{20}=\frac{143}{20}[/tex]

the volume of prism A is 144^3 if the base is 24^2 what is the height of prism A?

Answers

Answer

Height of prism A = 6 units

Explanation

The volume of a prism is given as the product of the area of a face that occurs on two sides of the prism and the distance between the two faces.

In the case of this face being a base, the volume of the prism is given as

Volume = (Area of Base) × (Perpendicular height)

Volume = 144 m³

Area of base = 24 m²

Perpendicular height = h = ?

Volume = (Area of Base) × (Perpendicular height)

144 = (24) × (h)

144 = 24h

We can rewrite this as

24h = 144

Divide both sides by 24

(24h/24) = (144/24)

h = 6 units

Hope this Helps!!!

5. Is X-1 a factor of x^5+2x^2-1?No, because f(1) = 2.Yes, because f(1) = 3.No, because f(1) = 0.Yes, because f(1) = 0.

Answers

Finding a factor of a polynomial

We want to find if x-1 is a factor of

[tex]f(x)=x^5+2x^2-1[/tex]

In order to verify that, we must know the last number of the synthetic division of the polynomial divided by x - 1. If it is zero then it is a factor, and if it is not zero then it is not a factor

If we replace x = 1 in the equation we will find that number:

[tex]\begin{gathered} f(x)=x^5+2x^2-1 \\ f(1)=1^5+2\cdot1^2-1 \\ f(1)=1^{}+2^{}-1 \\ f(1)=2 \end{gathered}[/tex]

Then the residual of the polynomial divided by x - 1 is 2, then x - 1 is NOT a factor.

Answer: A No, because f(1) = 2.

What is the length of side s of the square shown below?45°6S90°A. 2.B. 6C. 3D. 5.2E. 3.2F. .6

Answers

The diagram shows a square with one side marked as s, while the diagonal that cuts across measures 6 units.

The diagonal results in a right angled triangle with two sides measuring 45 degrees and one side measuring 90 degrees. Now that we have a right angled with one angle, and two sides (one is given as 6, and one is unknown), we now calculate side s as follows;

[tex]\begin{gathered} \cos 45=\frac{\text{adj}}{\text{hyp}} \\ We\text{ use the ratio for cosine because the sides shown are the} \\ \text{adjacent (between the right angle and the reference angle) and} \\ \text{hypotenuse (facing the right angle)} \\ \cos 45=\frac{s}{6} \\ \cos 45=\frac{1}{\sqrt[]{2}} \\ \text{Therefore,} \\ \frac{1}{\sqrt[]{2}}=\frac{s}{6} \\ \text{Cross multiply and you have} \\ \frac{6}{\sqrt[]{2}}=s \\ \text{Rationalize the expression and you have} \\ 3\sqrt[]{2}=s \\ \text{Therefore} \\ s=3\sqrt[]{2} \end{gathered}[/tex]

The correct answer is option E

What is the first step to solve this equation: 5m + 10= 7m + 4A: subtract 5m to both sides B: add 5m to both sides C: divide by 5 to both sides D: multiply by 5 to both sides

Answers

We are given the linear equation

[tex]5m+10=7m+4[/tex]

If we subtract 5m to both sides we get

[tex]5m+10-5m=7m+4-5m[/tex]

Simplifying

[tex]10=2m+4[/tex]

which is one step closer to the solution, therefore the answer is A.

Find the measure of each angle in the diagram. ​

Answers

Answer:

10y+7x+4+4×-22+3y+11

10y+3y+7x+4x+4-22+11

13y+11x-9

m = y2-yi=X2-X1Find the slope of the line that passesthrough these two points.(6,4)m = [?](2, -4)-

Answers

The slope between two points (x1,y1) and (x2,y2) is given by:

[tex]m=\frac{y_2-y_1}{x_2-x_1}[/tex]

In this case we have the points (2,-4) and (6,4), then:

[tex]\begin{gathered} m=\frac{4-(-4)}{6-2} \\ m=\frac{4+4}{4} \\ m=\frac{8}{4} \\ m=2 \end{gathered}[/tex]

Therefore, the slope is 2

Hi I just wanted you to check over my work to let me know if I did it correct

Answers

Answer:

Hello, Which part would you like to have checked?

I can't seem to make out your work from that of the assignment.

Step-by-step explanation:

Just let me know, here to help!

Graph the solution to the following inequality on the number line,(x + 6) (x-3) 20

Answers

If you have an equality of the form...

[tex](x+a)\cdot(x-b)\ge0[/tex]

The inequality can be writen as...

[tex]a\leq x\leq b[/tex]

I'll show you how it looks like on the graph

the graph indicates that x is any value less than or equal to -6 or greater than or equal to 3, which is the red area

[tex]\begin{gathered} x\leq-6 \\ x\ge3 \end{gathered}[/tex]

Solve the following absolute value inequality. Express your answer in interval notation.

Answers

Okay, here we have this:

We need to solve the following inequality, let's do it:

[tex]\begin{gathered} 3\mleft|5-y\mright|\le\: -6 \\ \mleft|5-y\mright|\le\: -2 \end{gathered}[/tex]

And considering that the absolute value cannot be less than zero, it means that the inequality has no solution in the set of reals.

Which of the following is the graph of the following system of equations? { 2x - 3y>12 {y< -1 x + 5 2

Answers

SOLUTION

The graph of the inequality

[tex]\begin{cases}2x-3y\ge12{} \\ y<-\frac{1}{2}x+5\end{cases}[/tex]

is shown below

The darkest part is the required region

Comparing with the options,

the answer is the graph below

-3v - 14> 3v+16 It’s for my final review and I’m so confused

Answers

The given inequality is

-3v - 14 > 3v + 16,

Add 3v on both side of the inequality

-3v -14 +3v > 3v +16 +3v

-3v +3v -14 > 3v + 3v + 16

0 - 14 > 6v + 16

Subtract 16 from both side of the equation,

-14 - 16 > 6v + 16 - 16

-30 > 6v + 0

Now subtract 6v from both side,

-30- 6v > 6v - 6v

-6v -30 > 0

Add 30 on both side,

-6v -30 + 30 > 30

-6v > 30

Divide both side by 6:

-6v/6 > 30/6

-v > 5

Multiply both side by (-1)

(-1) (-v) < (-1) 5

v < -5

Thus, v < -5

Answer : v < -5

Answer:

v < -5

Step-by-step explanation:

Add 14 to each side

[tex]-3v-14+14 > 3v+16+14[/tex]

Next, you need to simplify these additions by adding non-variable integers

[tex]16 + 14 = 30\\= 3v + 30\\= -3v > 3v + 30[/tex]

Now, subtract each side by 3v, the leading term in this equation.

[tex]-3v-3v > 3v+30-3v[/tex]

Again, we must simplify to get to the next step. This time we are dealing with a negative minus a negative. There on, things get trickier. In short words, a negative minus a positive is going to have a negative result, because both sides are treated as a negative plus a negative. In an equation, -a - b is the same as -a + -b.

[tex]-3v - 3v = -3v + -3v\\= -6v\\3v + -3v = 3v - 3v\\= 0\\= 0 + 30\\= 30\\= -6v > 30[/tex]

Next, we must multiply both sides by -1. The reason we're multiplying by a negative instead of a positive is because we are reversing the inequality.

[tex]= -6v\times -1 < 30\times 1[/tex]

Notice the change from greater than to less than.

IMPORTANT NOTE: A negative times a negative is a positive, a positive times a positive is also a positive. Lastly, a negative times a positive is a negative.

[tex]-6v\times -1 = 6v\\30\times -1 = -30\\= 6v < -30[/tex]

For the next step, we must divide by 6, because 6 can be divided by 6, and -30 can also be divided by 6 fairly.

[tex]\frac{6v}{6} < \frac{-30}{6}\\\frac{6v}{6} = \frac{6\times v}{6}\\Assume\:v=1!\\= \frac{6\times 1}{6}\\= \frac{6}{6}\\= 1\\= v[/tex]

[tex]\frac{-30}{6}\\\\(-a)/b = (-a/b)\\\\=-\frac{30}{6}\\= -5[/tex]

And therefore, v is less than -5.

If you need anything else, let me know!

Hope this helps!

What do the expanded form and a place- value chart tell you about a number such as 25,049?How are they alike and different

Answers

The expanded form of a number is written out in such a way that you would be able to see the math value of individual digits.

The number 25,049 which reads "twenty five thousand and forty nine," can be written in expanded form as follows;

2 x 10000 = 20,000

5 x 1000 = 5,000

0 x 100 = 0

4 x 10 = 40

9 x 1 = 9

So, as you can see the expanded form shows that the digit 2 (for example) in this number has a value of 20,000.

The place value chart groups the number in threes starting from the right to the left. That is, you count three numbers from the end (right hand side) insert a coma, and take the next three set of digits and so on. You can now tell the value of each digit by starting from the left (the begining) to the right (the end). Usually starting with billions, the chart now tells you the value of each digit. So in this question, you can read from left to right and by using the expanded form, you can tell the value of the 2, the 5, and so on till the last digit.

They are alike because the expanded form helps you determine the actual value of each digit without mistake, and the place value also tells you which number carries what value.

They are different because an expanded form gives you details of how each number gets its place value, while the place value chart simply tells you the value of each digit simply by arranging and inserting comas.

According to the diagram, an 8-foot-tall statue casts a shadow on the ground that is 15 feet in length. Based on this information, which trigonometric ratio has the value 8/15 ?A. cos CB. tan BC. cos BD. tan C

Answers

the right optio is tan C because...

[tex]undefined[/tex]

Use the Intermediate Value Theorem to show that the polynomial function has a zero in the given interval.

Answers

Given:

[tex]f(x)=10x^4-4x^2+5x-1;\lbrack-2,0\rbrack[/tex]

Using the intermediate value theorem,

[tex]\begin{gathered} f(x)=10x^4-4x^2+5x-1 \\ f(-2)=10(-2)^4-4(-2)^2+5(-2)-1 \\ f(-2)=160-16-10-1=133 \\ \text{and} \\ f(0)=10(0)^4-4(0)^2+5(0)-1=-1 \end{gathered}[/tex]

So, we have find value c between [-2,0].

[tex]\begin{gathered} f(x)=0 \\ 10x^4-4x^2+5x-1=0 \\ \Rightarrow x=-1\text{ it satisfies the equation} \\ \text{Also, -1}\in\lbrack-2,0\rbrack \end{gathered}[/tex]

It shows that, the above polynomial function has zero in the given interval.

Also, the value of f(-2) = 133

Yuson must complete 30 hours of community Service. She does two hours each day. Write a linear equation to represent the hours she has left after X days.

Answers

Yuson must complete 30 hours of community service.

She does two hours each day.

We are asked to write a linear equation to represent the hours she has left after x days.

We can write the following linear equation

[tex]30-2x=0[/tex]

Where 30 represents the total hours of community service that Yuson has to complete.

2 represents the hours she works each day.

x represents the days.

We can also solve this equation to find how many days will it take her to complete the community service.

GIVING 100 POINTS!!
1.) Angles A and B are supplementary. Determine the measure of angle A if the measure of angle B is 115.2°.

A) 244.8°
B) 64.8°
C) 25.2°
D) 11.5°

2. Find the sum of the interior angles of a 22-sided polygon.

A) 1,980°
B) 2,160°
C) 3,360°
D) 3,600°

HELP! ME PLS!! TWO QUESTIONS!

Answers

Answer:

Step-by-step explanation:

1. The answer is C because 180-115.2=64.8

2. The answer is D.

Please give me brainliest!

Answer:

1. C

2. D

Step-by-step explanation:

Because

Evaluate the following expression.x³ when x = 5

Answers

Given x^3, set x=5 and find the corresponding value, as shown below

[tex]\begin{gathered} x=5 \\ \Rightarrow x^3=5^3=5*5*5=25*5=125 \end{gathered}[/tex]Thus, the answer is 125

Hello, May I please get some assistance with this homework question? I posted an image below Q2

Answers

Solving (a)

The two functions we have are:

[tex]\begin{gathered} f(x)=3x+3 \\ g(x)=x^2 \end{gathered}[/tex]

We are asked to find the composite function:

[tex](f\circ g)(x)[/tex]

Step 1. The definition of a composite function is:

[tex](h\circ k)(x)=h(k(x))[/tex]

In this case:

[tex](f\circ g)(x)=f(g(x))[/tex]

This means to plug the g(x) expression into the value of x of the f(x) function.

Step 2. Substituitng g(x) as the value for x in f(x):

[tex](f\circ g)(x)=f(g(x))=4(x^2)+3[/tex]

Simplifying:

[tex](f\circ g)(x)=\boxed{4x^2+3}[/tex]

Step 3. We also need to find the domain of this composite function.

The domain of a function is the possible values that the x-variable can take. In this case, there would be no issues with any x value that we plug as the x-value. Therefore, the domain is all real numbers.

The domain of fog is all real numbers.

Answer:

[tex](f\circ g)(x)=\boxed{4x^2+3}[/tex]

The domain of fog is all real numbers.

What is the prime factorization of 72?
A. What is the prime factorization of 72?
A.

Answers

Answer:

2 3 ⋅ 3 2

Step-by-step explanation:

Answer:23·32

Step-by-step explanation:72 divided by 32 will give you 23 and 32 multiplied by 23 equals 32

Select the values that make the inequality m≥7 true.
(Numbers written in order from least to greatest going across.)

Answers

Step-by-step explanation:

every number greater than or equal to 7.

just as the expression says.

are we taking only about integer values ?

so,

7, 8, 9, 10, ... +infinity

if we aim for radical or real values, then starting with 7 everything between these numbers.

the interval definition is

[7, +infinity)

please note the different brackets.

"[" or "]" means the interval end value is included.

"(" or ")" means the interval end value is excluded.

which is the normal thing for infinity, because infinity is only a concept and never a number. so, it cannot be included.

This is a practice assessment that will not be graded! Just need help finding this answer to understand it overall

Answers

The general structure of the equation of a circle is:

[tex](x-h)^2+(y-k)^2=r^2[/tex]

Where

h is the x-coordinate of the center of the circle

k is the y-coordinate of the center of the circle

r is the radius of the circle.

Note that the equation has minus signs inside the parentheses, this means that the sign of the coordinates is the opposite as the one shows on the equation.

The first step is to identify the coordinates of the center of the circle in each equation as well as the radius:

Equation 1:

[tex](x-3)^2+(y+2)^2=9[/tex]

The x-coordinate of the center is the value inside the first parentheses: h= 3

The y-coordinate of the center is the value inside the second parentheses: k= -2

[tex]center\colon(3,-2)[/tex]

To determine the radius you have to calculate the square root of the last number of the equation:

[tex]\begin{gathered} r^2=9 \\ r=\sqrt[]{9} \\ r=3 \end{gathered}[/tex]

Use the same logic for the other three equations:

Equation 2:

[tex](x-3)^2+(y-2)^2=16[/tex]

h=3

k=2

[tex]\text{center:(3,2)}[/tex]

Radius:

[tex]\begin{gathered} r^2=16 \\ r=\sqrt[]{16} \\ r=4 \end{gathered}[/tex]

Equation 3

[tex](x+3)^2+(y+2)^2=16[/tex]

h=-3

k=-2

[tex]\text{center:(-3,-2)}[/tex]

Radius:

[tex]\begin{gathered} r^2=16 \\ r=\sqrt[]{16} \\ r=4 \end{gathered}[/tex]

Equation 4

[tex](x-3)^2+(y-2)^2=9[/tex]

h=3

k=2

[tex]\text{center:(3,2)}[/tex]

Radius:

[tex]\begin{gathered} r^2=9 \\ r=\sqrt[]{9} \\ r=3 \end{gathered}[/tex]

Next, you have to determine the center and the radius of each graph:

Circle 1:

Has a radius with a length of 4 units and center (3,2), the equation that corresponds to this circle is the second equation.

Circle 2:

Has a radius with a length of 4 units and the center at (-3,-2), the equation that corresponds to this circle is the third equation.

Circle 3:

Has a radius with a length of 3 units and a center at (3,2), the equation that corresponds to this circle is the fourth equation.

Circle 4:

Has a radius with a length of 3 units and center at (3,-2), the equation that corresponds to this graph is the first equation.

What is the definition of function?Hos inputs andoutputsInputs haveEvery input hosonly ONE outputxrches andy-wolvesdifferent outputsevery time

Answers

The definition of function is

Find the surface area of the giving prism round to the nearest 10

Answers

The surface area of the given prism is the sum of areas of all sides.

From the given figure, we have :

2 Triangles with a base of 9 ft and a height of 7.6 ft

1 rectangle with a length of 13 ft and a width of 10 ft

1 rectangle with a length of 13 ft and a width of 8 ft

1 rectangle with a length of 13 ft and a width of 9 ft

The formula for the area of a triangle is :

[tex]A=\frac{1}{2}\times Base\times Height[/tex][tex]A=\frac{1}{2}\times9\times7.6[/tex][tex]A=34.2[/tex]

Since there are two triangles, the total area of the triangle is :

[tex]A=2\times34.2=68.4[/tex]

The formula for the area of the rectangle is :

We can add the three triangles together.

[tex]A=(13\times10)+(13\times8)+(13\times9)[/tex][tex]A=130+104+117[/tex][tex]A=351[/tex]

Now we have the areas of the sides, take the sum of these areas to find the surface area.

[tex]\text{Surface Area = 68.4 + 351}[/tex][tex]\text{Surface Area = 419.4 ft\textasciicircum{}2}[/tex]

15 percent of a certain company's life insurance policy holders are smokers. For each nonsmoker the probability of dying during the year is 0.011. For each smoker the probability of dying during the year is 0.04. Find the probability that a policy holder who died last year was a smoker.

Answers

[tex]P=(Percentage_{\text{Smoker Policy Holders}})(Probability_{\text{Smokers Dying}})[/tex]

Percentage of Smoker Policy Holders = 15% / 100% = 0.15

Probability of Smokers Dying = 0.04

Let's substitute the values to the equation, we get,

[tex]\text{ P = (0.15)(0.04) = 0.006}[/tex]

The probability that a policy holder who died last year was a smoker = 0.006

find the exact value of cosine Pi / 3 express your answer with a rational denominator

Answers

it is given that,

the expression is

cosine Pi/3

we know that

so,

[tex]\cos \frac{\pi}{3}=\cos \frac{180}{3}=\cos 60=\frac{1}{2}[/tex]

thus, the answer is 1/2

What is the measure of angle J in the triangle below? *Hint: Law of Sines*

Answers

SOLUTION:

Using the sine rules;

The equations developed;

[tex]\frac{15}{sin102}=\frac{12}{sinJ}[/tex]

Making sinJ the subject;

[tex]\begin{gathered} sinJ=\frac{12sin102}{15} \\ sinJ=0.7825 \\ J=51.49^o \end{gathered}[/tex]

Thus, the angle is 51.5 degrees.

train moves at a constant speed of 8 miles every 6 minutes. Fill in the table below to show how far the train travels according to differentmounts of time.Time (minutes)Distance (miles)31560

Answers

we have the following:

speed 8 miles every 6 minutes:

[tex]s=\frac{8}{6}=1.34[/tex]

therefore, the speed is 1.34 mi/m, now to complete it would be:

[tex]\begin{gathered} d=s\cdot t \\ d1=1.34\cdot3=4 \\ d2=1.34\cdot15=20 \\ d3=1.34\cdot60=80 \end{gathered}[/tex]

therefore, the answer is:

Time (minutes) Distance (miles)

3 4

15 20

60 80

write an equation in slope intercept form of the line that is perpendicular to the line Y equals 1/4 x -9 and passes through 1, 1

Answers

For perpendicularity

[tex]\begin{gathered} \text{slope of line 1 =}\frac{-1}{slope\text{ of line 2}}_{} \\ \text{slope of line 1 = m}_1 \\ \text{slope of line 2 =m}_2 \end{gathered}[/tex][tex]\begin{gathered} \text{Hence,} \\ m_1=\frac{-1}{m_2} \end{gathered}[/tex][tex]\begin{gathered} \text{From the question} \\ m_1=\text{ }\frac{1}{4} \end{gathered}[/tex][tex]undefined[/tex]

ine TrackerWhat additional piece of information is needed in order to say thatthese two triangles are congruent by AAS postulate?BO BC DEO AB DEO BC EFO AB DF

Answers

Answer

Option B is correct.

AB ≅ DE

Explanation

The key to two triangles being similar according to AAS is that they have two angles and an excluded side in common.

An excluded side does not reside between the two congruent angles.

So, for these two triangles to be congruent according to AAS,

Angle C = Angle F

Angle B = Angle E

And

Side AB ≅ Side DE

Hope this Helps!!!

Other Questions
A 4.3-kg bat is traveling 21 m/s just before it strikes a 2.5-kg baseball thats moving 37 m/s. After the collision, the bat travels in the same direction at 19 m/s. What is the baseballs speed just after impact? AT 6 AM today, you purchased 1 MW of electricity contract for 12 PM at a price of 100 pounds/MWh. Two hours later, the forecast for solar generation for 12 PM has changed from 4 GW to 4.5 GW. The market is currently bid at 95 pounds/MWh and offered at 105 pounds/MWh. What would you do, and why? Please answer logically, stating all assumptions. Note that no additional research is needed. If the spinner is spun, what is the probability that the spinner will land on a multiple of 4? Choose ALL answers that describe thequadrilateral DEFGit DE FG,EF | GD, diagonal DF = 16, and diagonalEG=16. i really really need help on this qustion Part 311Use the relationships in circle O to find the missing measures in circle S.1 pointIf WR = 12 units and MN = 8 units, determine MW.Type your answerwMRNePrestipus Hello! I need some assistance with this homework question for precalculus, please?HW Q27 what is the lcm of 25 and 37? What is the congruence correspondence, if any, that will prove the given triangles congruent?A. SASB. AASC. noneD. ASA Precalculus:Consider the right triangle where a = 3mi and alpha = 45 degrees Find an approximate value (accurate up to three or more decimals) of each of the following. Give the angle in degrees. which system of equations is better to start up to solve using the subsition method or the elemination method? What is the measure of an interior angle of a regular decagon?1814418036 At the city museum, child admission is $5.90 and adult admission is $9.40. On Saturday, 170 tickets were sold for a total sales of $1332.00. How many adulttickets were sold that day?Number of adult tickets: What is the real part of 4 5i? 54-5i-5 i have no idea how to do this please help In the expansion of (2a+4b)^8, which of the following are possible variable terms? Explain your reasoning? Consider the diagram and angle measures shown below.m1 = (3x +25)m2 = (7x+5)m3 =(-2x + 70)4322 || 7What is the value of m3 ? A painting sold for $216 in 1977 and was sold again in 1985 for $467. Assume that the growth in the value V of the collectors items was exponential. A. Find the value k of the exponential growth rate. Assume v=216 K=? I need to find out number 51.Can you help me ? How is a number related to its opposite?